2015 A Level H2 Physics (9646) Paper 1 Suggested Solutions

All solutions here are SUGGESTED. Casey will hold no liability for any errors. Comments are entirely personal opinions.

1.A
2.D
3.C
4.A
5.C
6.A
7.D
8.C
9.A
10.C
11.B
12.A
13.D
14.A
15.A
16.A
17.C
18.A/ C ;I am not sure of Q18 though. Cos in reality, there is a phase change of pi on reflection, in which case, the answer shd be C. But phase change is not in the syllabus, then the answer should be A. so I dun know what Cambridge wants..:-(
19.C
20.B
21.C
22.D
23.B
24.B
25.A
26.B
27.B
28.B
29.D
30.B
31.B
32.D
33.B
34.C
35.C
36.C
37.C
38.A
39.D
40.B

Note to all: Casey will not respond to most of the comments as he is busy. You may contact him by SMS at  +65 9474 5005 if you have a burning question.

Feel free to explain the answers, if you are confident. Many thanks.

Showing 148 comments
  • Lolister
    Reply

    gg.com this is bloody hard

  • Student
    Reply

    Hi sorry, may I know is it possible to get the H1 Physics Paper 1 answers up too? I know it will only benefit a small group of us but it’ll be greatly appreciated. Thank you! 🙂

  • Emelia
    Reply

    Hi! May I ask if you guys be posting the ans for H1 physics paper 1?

  • Hi
    Reply

    Hi, may I know where is the answer? What is the rough (just estimation will do) mark to get A, B and C based on bell curve?
    Tq

  • ...
    Reply

    Can you upload H1 Paper 1 answer also? Thanks A Lot!

  • BTeam
    Reply

    I LOST THE FIRST MARK wasted:’). It’s mol. Not charge. You can search it up

    • Anonymous
      Reply

      Charge is Current*Time, time already has second in it so this is why it’s not the answer.

  • Failure
    Reply

    I think I can fail this paper 🙁

  • fml
    Reply

    I think I can get like less than 20?????? Am I the only one who finds it tough cause my friends are saying that they can get full marks…

    • student
      Reply

      It is difficult but I dont think you will fail!

      • Anonymous
        Reply

        I got about 28 based on preliminary answers from classmates…that’s bad 🙁

        • student
          Reply

          I got 35!

          • studentrightoutoftheexam

            same i got 35 too 🙂

  • Student
    Reply

    Anyone wants to post their ans? haha

  • ywj
    Reply

    Can you do for h1 physics too?

  • LALALALALA
    Reply

    GGWP CAMBRIDGEEEEEE MUACKSSSSS

  • Cambridge
    Reply

    Anyone got a different answer for Q28?

    • student
      Reply

      28 was tested in one of the past year alevels so it should be correct

  • studentxyz
    Reply

    Can do for h1 physics paper 1 too?

  • anon
    Reply

    How much u guys get?

    • student
      Reply

      29, including a for qn 18

  • Student
    Reply

    Why is 36 C and not B? Doesn’t it have to be the same direction?

    • KS Teng
      Reply

      Cos the momentum shd occur in the same direction as the velocity.. Which is horizontal.
      But diffraction means it spreads vertically..
      – Casey

      • Student
        Reply

        Can apply uncertainty principle with different direction?

      • student
        Reply

        hi, isn’t it A though? I did a similar question and the answer was A

        • Student
          Reply

          Ya I think A is correct but I put B because I thought the arrow before the screen means it has not touched the screen. But A should be the answer because it is definitely correct.

        • student
          Reply

          imo I thought x should be at the slit and p at the screen eh but no such option HAHA.

          • Student

            must be simultaneous measurement

          • xavierwwj

            Answer is A.

            Parallel to the slit, the deltaX is small and limited to the slit width, whereas perp to the slit, the position can vary to a large extent, as there are no obstacles so deltaX is significant. As such momentum uncertainty occurs parallel to the slit, as noticed by the greater spreading in the interference pattern. While the momentum uncertainty occurring perp to the slit would be minute.

  • answer
    Reply

    were the answers stolen?

    • KS Teng
      Reply

      No. We aren’t trying to compete to be first, to be honest. I hope you understand that we all have personal commitments and realise that this is not our obligation, nor is it your entitlement. We do not derive any marginal utility doing this, only losing time on our side.

      And this set of answers, I helped Casey to upload them. So there is lag time on my side, as I have a full-time job and was held up too. So my bad for the delay.

      Also, the reason why your comments were not shown was because you linked a link and copy-pasted it a few times, which is why it picked you up as spam. I “un-spammed” you. We set the comment setting rather high, to remove profanities and external links actually. I hope you understand. Thanks

      • studentrightoutoftheexam
        Reply

        To all those tutors working at The Culture, thank you so much for putting in the effort to help us with the answers and guide us through our As even though we do not attend classes at your tuition centre. A really big big thank you ;):)

        • KS Teng
          Reply

          hi, its a pleasure. Most of us are not full-time tutors though we only teach in the newtonapple. The purpose of this was to facilitate discussion, and previous feedbacks from ex-students were that they were able to learn from the mistakes made by us and the students of the year. So it actually helps learning for everybody. This site isn’t really about tuition actually, as we do shall undergraduate & post-graduate stuffs here too. Some students had asked us regarding postgraduate or undergraduate studies, and we are more than willing to share a bit. Lastly, we love your own field of studies and are glad we can help ourselves and you learn from the mistakes. 🙂

  • student
    Reply

    Mr Teng why is question 6 A and not C?

    • Casey
      Reply

      This is Casey.. Cos they asked for additional weight.. And the barge was always floating, whether it was over the bridge or not. So, no additional force.. Hope that helps..:-)

  • student
    Reply

    Anyone getting >30?

    • whythisyearliddat
      Reply

      31 only..

      • Happy sad
        Reply

        I got 31 also :/ But it was expected to be difficult cos paper 3 was quite manageable. Don’t be disheartened! Bellcurve won’t deviate so much I guess? Enjoy guys! It’s the end of As (for most)!

    • Student
      Reply

      37 I think

    • wo-shi-ace
      Reply

      35 😀

      • Troll
        Reply

        I got 40 if 36 is A!

  • answer
    Reply

    q18 is C CONFIRM

  • student
    Reply

    Mr Teng why is the answer for Question 4 A and not B?

    • Casey
      Reply

      This is Casey.. Cos the mass decreases continuously . Since propelling force is constant, the acc increases.

  • student
    Reply

    Guys how much did you and most of your friends get out of 40 for this paper?

    • student
      Reply

      Mostly around 30

      • anon
        Reply

        Upper tier jc?

        • Happy sad
          Reply

          This guy called “student” has been trolling on all forums will all the fake bellcurve and stuff so avoid seeking comments from non-professionals..

          • student

            hey but i only got around 29 for this paper so how is that trolling?

  • Student
    Reply

    Why is 33 B?

  • Student
    Reply

    I feel that the ans for 15 should be B. Why is B wrong?

    • Casey
      Reply

      Cos ePE = \frac{1}{2}k x^2.. So the increase in the ePE cannot be equal when the extension doubles..

      • student
        Reply

        but the question says from the equilibrium position tho

        • Casey
          Reply

          Ya, so you notice that the change in ePE is 1.9 J from highest point to eqm position.. So from highest point to lowest point, the change in ePE cannot be merely doubled cos of the x^2

    • LOL EASY EXPLANATION
      Reply

      u try adding up all the powers together.
      by COE, total energy should remain same.
      if u do option B, +1.9-3.7 is -1.8 NOT 1.8 so cannot

      • student
        Reply

        gpe and epe are given as change from centre position so its not COE

        • student
          Reply

          if you say its the change from the top position then i feel A is the correct answer

          • student

            I agree with him, isn’t it from equilibrium? Then it should be positive change of the same magnitude

          • Student

            I think the spring is not at its natural length at equilibrium point because the mass itself will cause an extension

          • student

            Lets say at eqm extension is e, and amplitude of SHM is a.

            EPE @
            top: 0.5k(e-a)^2
            eqm: 0.5ke^2
            btm: 0.5k(e+a)^2

            if you let eqm EPE =0 (aka deducting 0.5ke^2 from top, eqm and btm),

            top: 0.5k(e-a)^2 – 0.5ke^2 = kea – 0.5ka^2
            eqm: 0
            btm: 0.5k(e+a)^2 – 0.5ke^2 = kea + 0.5ka^2

            as you can see, EPE@btm is not equal to negative of the EPE@top

            hope this helps convince you 🙂

  • Student
    Reply

    How can 24 be B? I thought it was similar to one of the tys qn

    • Student
      Reply

      Greater V means greater force and hence acceleration. So vertical speed increases

    • anon
      Reply

      The tys qns is for magnetic field

  • Student
    Reply

    Why isn’t 29 answer B?

    • Casey
      Reply

      Cos X is not ohmic, so it’s resistance will change with V.. So need to refer to graph for its specific R at the particular V and I

      • Student
        Reply

        Why can’t I find the individual resistance of X and Y, then I arrange them in parallel.Then from there I calculated the effective resistance of the circuit. And the emf/I should give the effective resistance.

  • hi
    Reply

    can you upload h1 physics p1 as well

    • Casey
      Reply

      Sorry.. I dun have the paper.. My son is Also taking his A levels this year and he does H2..

      • hi
        Reply

        is there a way i can show you the questions? i cannot seem to find answers from anywhere else

        • KS Teng
          Reply

          you can try whatsapp-ing him and ask if he is free and send the images. His number is above. 🙂

  • ....
    Reply

    can you upload h1 physics p1 as well

    • Casey
      Reply

      Oh.. I din look at the link.. But maybe you are right.. Thanks for the input.. Cheers..:-)

      • Student A
        Reply

        Yup. I believe the answer is A. I have seen this question in Cambridge International A levels before.

        • Student
          Reply

          If A is the answer then what about B? Aren’t they similar but just along different points of the path of electron?

          • Student A

            B cannot be accepted as once the electron arrived at the screen, there will not be any associated uncertainty to it. The reason is simply because any system that is being observed will cause the wave function of the electron to collapse. So in this case, the uncertainty associated with p and x should be along the slit width and not at the screen since the patterns on the screen can be observed.

          • Student

            I thought the arrow is before the screen means it has not reached the screen yet lol

  • aaa
    Reply

    Is 27 a bad score?

  • jen
    Reply

    Why 36 not D?

    • Student
      Reply

      Must be simultaneous measurement and same direction

    • Student
      Reply

      What about B?

    • Shawn
      Reply

      Agreed. The momentum and postional uncertainity must be in the same direction. This is one of the assumptions of the calculation. Furthermore there needs to be a vertical momentom for diffraction to occur. If not no diffraction will occur and the light is simply a bright spot

  • student
    Reply

    How many marks out of 40 did you guys get for this paper?

  • Student
    Reply

    For qn 15, I got B bcos at the bottom most position, mg=kx, from there I calculated k=163.5. Then at the bottom position EPE=(1/2) (k) (x^2)=(1/2) (163.5) (0.15^2) I got B . Is that wrong?

    • Casey
      Reply

      But x shd be the total extension.. So from eqm downwards the total extension shd not be just 0.15m..

  • Stud
    Reply

    What is the more likely answer for 18? A or C?

    • Student A
      Reply

      Most likely it’s C as the physics concept behind it is correct.

  • Student
    Reply

    I also feel that the answer should be A for Q36!

  • Anonymous
    Reply

    Why isnt the answer for question 4, C?

    It is similar to this qn… http://physics.stackexchange.com/questions/91110/increase-in-velocity-by-loss-of-mass

    • Casey
      Reply

      That question is a bit diff cos there is no external driving force.. So linear momentum is conserved. As in the sand’s X speed while in the trolley and when it dropped out is the same.. So by PCoLM, the
      Trolley’s speed remain constant..

  • Student 3x
    Reply

    Just to clarify…
    for Question 5, why isn’t it B?
    Taking rightwards as positive , and since both spheres have same mass,
    mu1 – mu2 = mv1 + mv2
    m(u1-u2) = m(v1+v2)
    u1-u2=v1+v2 which is B? What’s wrong with this working ?

    • Student 3x
      Reply

      Also for Qn 4,
      since propelling force horizontally is constant, and the water runs out at a constant rate, shouldn’t it accelerate uniformly? since water is running out at a constant rate

      • Student
        Reply

        Constant F with decreasing m. Hence a increases. Accelerate uniformly means a constant

        • Student 3x
          Reply

          I thought accelerate uniformly means like increasing at 2ms-2 constantly! haha thanks though!

      • student
        Reply

        For elastic collision of 2 objects with same masses, they will exchange velocities. Hence velocity of v1 should actually be negative, and hence cannot use COM. Ans is C is definitely correct.

    • Student A
      Reply

      There is nothing wrong with your workings for question 5. But, it does not show that the collision above is elastic. It is merely a statement of the conservation of linear momentum. So, the best answer is C as it shows that the collision is indeed elastic (can be derived easily from conservation of KE)

      • Student 3x
        Reply

        ahh I was contemplating on which to put during the paper, regardless thanks for the explanation!

  • h1
    Reply

    please upload the h1 paper also thanks!!

  • Hi
    Reply

    Hi, How many overall mark tentatively to get B or C? Roughly only. Based on bell curve. It seems my comments got deleted 🙁

    • student
      Reply

      B should be 60-65 range because mcq is difficult

  • student
    Reply

    18 should be C bc i rmb got learn before, and can be explained by N3L at the fixed point, and 2010 q21 also got reflected wave qn also (but that one on stationary waves).

  • taka
    Reply

    my guess wuld be ard 60-70, which is the range for most papers like chemistry or biology. maybe slighly less than 70 cos tats like too xtreme. so ard 60-65??

  • hi
    Reply

    hi. Thanks for uploading the answers. Rmb guys this mcq is just 20%, so for every 2 wrong answers u lose 1%. So if u get 30 and your friend gets 35, u only lose him by 2.5% which is not that bad. Since its over lets just take a break and rejoice! Enlisting dam soon in 4JAN tho.. Good luck everyone and all the best for your future endeavours

    • KS Teng
      Reply

      Very true. 🙂
      Enjoy your 2 years of break!

  • Mark
    Reply

    Hi, where can I get a hold of all the question papers for H2 physics, paper 1, paper 2 and paper 3? I would like to practice them immediately as I am private candidate.

    Thanks for your help

    • taka
      Reply

      erm… im pretty sure that u can buy the yearly tys frm the Popular. shuld have all 3 papers

      • Mark
        Reply

        I need the 2015 paper for practice, where can I get it immediately?

        • taka
          Reply

          lol. within such a short period of time. i dun tink u can get it on9, unless u have frens who r taking the A levels

          • KS Teng

            I highly doubt you can find it. Maybe only paper 1 for physics since the rest are collected back. Most scripts have their own bar code for copyright reasons. So you can only have papers that were allowed to bring home like Math P1 & P2, Physics P1, Chem P1 & P3. Pretty much those that we were able to do.

  • StUDENT
    Reply

    Is 28 good enough for this paper

    • PPP
      Reply

      I got almost the same as you don’t worry this paper is really really hard so ya don’t think too much and enjoy your holidays

  • jamie
    Reply

    hi, what’s the A and B grade overall range for H2 physics?

    • student
      Reply

      A ~ 75 B~65

  • jamie
    Reply

    hi what are you guy’s estimated overall score? mine is only 66…pretty worried.

  • 9647WHYULIKETHIS
    Reply

    hi sir, please upload H1 physics P1 too 🙂

  • jen
    Reply

    Is below 70 an A possible for this paper?

    • student
      Reply

      not possible A for physics is usually higher than chemistry ~75

  • Student 12
    Reply

    I wrote C for qns 4. I was choosing between A and C the whole time. For A to work, the truck had to initially be accelerating, so the loss in mass would lead to an increase in acceleration, Fnet=ma>0. But if it was initially moving at a constant speed, where Resistive force = Propellant force, a change in mass will not increase acceleration as Fnet = ma = 0. I chose C because it says it “continues at a constant speed”. I inferred from the word ‘continued’ and assumed that it was initially at constant speed. I do hope i went wrong somewhere so that i can learn from this. By the way answer for 36 is definitely A, my senior in Cambridge checked with the physics professor already. Momentum has to be in the same direction as the delta X or this conservation law will not hold. Just because vertical velocity is 0 in the question does not mean that the vertical velocity cannot be uncertain. It can be 0 +- 5. That’s why it even diffracts, it’s because when you close the slits, delta X becomes smaller and delta P increases, hence the momentum vertically is more uncertain, which is why divergence even takes place at all. This is universally the reason for diffraction. Check out Veritasium’s video on the uncertainty principle for this! Anyway, what’s more important is that we learn something new everyday! Someone please correct me on qns 4, thanks!

    • Student
      Reply

      For 4, because water is running out, there is a continual decrease in mass hence acceleration is increasing

      • Student 12
        Reply

        But how do you even know that it was accelerating in the first place?

        • Student
          Reply

          Doesn’t have to be accelerating at first. Even if acceleration is 0 initially it will still increase.

          • Student

            You have to assume whatever resistive force is constant. But they didn’t even mention any resistive force so just assume don’t have. Don’t have to think so much haha

          • Student 12

            It intuitively makes sense to me that it does not have to be initially accelerating. This is my guess for the reason. Although the equation “Fforward-Fresistance = ma” is not technically wrong. I think that this cannot be explained by such simple maths and we need a more intuitive explanation. Thus let us assume that Fresistance is just constant. Hence we can add this constant directly to Fforward. Now let’s equate Fforward = ma. Since Fforward is constant, m decreases and a increases [Which is the very simple way of doing this question without having my convoluted thoughts XD]. Initially the Fforward was used to overcome the resistance, thus a lower mass lowers the intertia of the system, thus the Fforward can now “afford to” overcome resistance and have a net rate of change of momentum due to the inertia of the body thus it accelerates non uniformly. Thus even if net force was zero initially, it is still possible to accelerate without changing the forward force. This is my guess. But good point, there may not even be resistance. Either way i think i kind of managed to explain how initial constant speed can lead to the answer A. Thanks!

  • student
    Reply

    Hi, why is the answer for question 11 B and not C?

    • student
      Reply

      It’s okay, I realised where’s my careless mistake already

  • student
    Reply

    How many marks out of 40 did you guys get for this paper

    • Student 12
      Reply

      Arnd 36/37?

  • student
    Reply

    How many marks out of 40 did you guys get for this paper 3?

    • Student
      Reply

      37

  • 8866/01
    Reply

    hi sir can you upload H1 physics too? im really worried because it is quite difficult

  • student
    Reply

    How many marks out of 40 did you guys get for this paper 1?

    • Student
      Reply

      Why keep asking same question?

  • Joelle
    Reply

    Is it possible for A grade to be arnd 70marks overall?

  • student
    Reply

    How many marks out of 40 did you get for this paper?

  • STUDENT B
    Reply

    Can u put H1 physics paper 1?

  • student
    Reply

    Mr Casey, can you explain to me why the specific heat capacity for an ideal gas when measured at constant volume of the gas is different from that when measured at constant pressure of the gas?

    • KS Teng
      Reply

      Cos for constant pressure, there is a change in volume.. Which means work need to be done against intermolecular forces of attraction to separate the molecules and also on the environment in prefer to push back the atmosphere to create room for expansion.. Hence more heat is required..
      – Casey

  • student
    Reply

    Mr Teng can you explain why the specific heat capacity for an ideal gas when measured at constant volume of the gas is different from that when measured at constant pressure of the gas?

    • KS Teng
      Reply

      Cos for constant pressure, there is a change in volume.. Which means work need to be done against intermolecular forces of attraction to separate the molecules and also on the environment in prefer to push back the atmosphere to create room for expansion.. Hence more heat is required..
      – Casey

  • student
    Reply

    Hi Mr Teng,what is the origin of the energy of sound waves, is it the sound signal generator?

  • student
    Reply

    Mr Casey what is the origin of the energy of sound waves? Is it the sound signal generator?

  • student
    Reply

    Hi Mr Teng and Mr Casey, what is the origin of the energy of sound waves? Is it the sound signal generator?

  • student
    Reply

    Hi Mr Teng and Mr Casey what is the origin of the energy of sound waves? Is it the sound signal generator?

Leave a Reply to student Cancel reply

Contact Us

CONTACT US We would love to hear from you. Contact us, or simply hit our personal page for more contact information

Not readable? Change text. captcha txt
0

Start typing and press Enter to search